Difference between revisions of "2016 AMC 10A Problems/Problem 4"

Line 7: Line 7:
 
==Solution==
 
==Solution==
  
The value, by definition, is <cmath>\frac{3}{8}-\left(-\frac{2}{5}\right)\lfloor{\frac{3}{8}*\frac{-5}{2}\rfloor=-\frac{15}{16}}=\frac{3}{8}-\frac{2}{5}=\boxed{\textbf{(B) } -\frac{1}{40}.}</cmath>
+
The value, by definition, is <cmath>\frac{3}{8}-\left(-\frac{2}{5}\right){\lfloor{\frac{3}{8}*\frac{-5}{2}\rfloor}=-\frac{15}{16}}=\frac{3}{8}-\frac{2}{5}=\boxed{\textbf{(B) } -\frac{1}{40}.}</cmath>
  
 
==See Also==
 
==See Also==
 
{{AMC10 box|year=2016|ab=A|num-b=3|num-a=5}}
 
{{AMC10 box|year=2016|ab=A|num-b=3|num-a=5}}
 
{{MAA Notice}}
 
{{MAA Notice}}

Revision as of 19:35, 3 February 2016

Problem

The remainder can be defined for all real numbers $x$ and $y$ with $y \neq 0$ by \[\text{rem} (x ,y)=x-y\left \lfloor \frac{x}{y} \right \rfloor\]where $\left \lfloor \tfrac{x}{y} \right \rfloor$ denotes the greatest integer less than or equal to $\tfrac{x}{y}$. What is the value of $\text{rem} (\tfrac{3}{8}, -\tfrac{2}{5} )$?

$\textbf{(A) } -\frac{3}{8} \qquad \textbf{(B) } -\frac{1}{40} \qquad \textbf{(C) } 0 \qquad \textbf{(D) } \frac{3}{8} \qquad \textbf{(E) } \frac{31}{40}$

Solution

The value, by definition, is \[\frac{3}{8}-\left(-\frac{2}{5}\right){\lfloor{\frac{3}{8}*\frac{-5}{2}\rfloor}=-\frac{15}{16}}=\frac{3}{8}-\frac{2}{5}=\boxed{\textbf{(B) } -\frac{1}{40}.}\]

See Also

2016 AMC 10A (ProblemsAnswer KeyResources)
Preceded by
Problem 3
Followed by
Problem 5
1 2 3 4 5 6 7 8 9 10 11 12 13 14 15 16 17 18 19 20 21 22 23 24 25
All AMC 10 Problems and Solutions

The problems on this page are copyrighted by the Mathematical Association of America's American Mathematics Competitions. AMC logo.png